Hardware Upgrade Forum

Hardware Upgrade Forum (https://www.hwupgrade.it/forum/index.php)
-   Scienza e tecnica (https://www.hwupgrade.it/forum/forumdisplay.php?f=91)
-   -   [Official Thread]Richieste d'aiuto in MATEMATICA: postate qui! (https://www.hwupgrade.it/forum/showthread.php?t=1221191)


-Ivan- 30-11-2012 15:46

Trovare il punto più lontano da una serie di punti all'interno di un cerchio
 
Ciao a tutti, ho un problema che nasce per motivi informatici ma che è prettamente matematico.

Mi trovo in questa condizione:
lavoro con uno spazio 3d, ma penso per ora possa approssimare ad uno bidimensionale per capire il concetto.
Ho un'area circolare ed, all'interno di questa, dei punti di cui ho le coordinate, dovrei trovare un nuovo punto, non presente all'interno del cerchio, il più distante possibile dagli altri punti ma non non esca da esso.

E' possibile in un modo non troppo complesso risolvere questa cosa?
Iuto! :mc:

zanardi84 06-12-2012 09:10

Dimostrazione di un limite notevole
 
Ho un limite notevole di cui voglio imparare la dimostrazione, ma non capisco un passaggio che evidenzierò.

Riporto tutto:

lim x->0 [((1+x)^k)-1]/x

che fa k.

Mi viene detto di prendere tutto il numeratore e di porlo uguale a y (cambio di variabile, e fin quì tutto ok)
y = [((1+x)^k) -1] . Se calcolo il limite ottengo il nuovo valore di tendenza per la y che è ancora 0, per x->0.

da cui: (1+x)^k = y+1.

Mi fa passare ai logaritmi.
kLog(1+x) = Log(y+1) (A)

E adesso arriva il passaggio che non capisco.
Quando cambio la variabile dovrei poi calcolare dall'equazione che ho indicato con (A) il valore di x da sostituire nella funzione per averla tutta in y.

Mi ritrovo invece

lim per x->0 e y->0 (kLog(1+x)/x)*(y/(Log(1+y))) da cui poi svolge i calcoli, applicando il prodotto dei limiti per ottenere k.

Che ha combinato in quel passaggio?

Grazie.

Grazie.

ChristinaAemiliana 06-12-2012 20:08

Mah, sembrerebbe un passaggio molto terra terra...dovrebbe aver sostituito il numeratore con y e poi moltiplicato il tutto per kln(1+x)/ln(1+y) che vale 1 per l'equazione che hai chiamato (A), ossia per come hai definito y, e infine riordinato in modo da separare le variabili. Da lì in poi è semplice, separi il limite in un prodotto di limiti e ti trovi con due limiti notevoli identici che fanno 1, percui il risultato resta k. :boh:

Amsirak 26-12-2012 22:38

Quote:

Originariamente inviato da -Ivan- (Messaggio 38633113)
Ciao a tutti, ho un problema che nasce per motivi informatici ma che è prettamente matematico.

Mi trovo in questa condizione:
lavoro con uno spazio 3d, ma penso per ora possa approssimare ad uno bidimensionale per capire il concetto.
Ho un'area circolare ed, all'interno di questa, dei punti di cui ho le coordinate, dovrei trovare un nuovo punto, non presente all'interno del cerchio, il più distante possibile dagli altri punti ma non non esca da esso.

E' possibile in un modo non troppo complesso risolvere questa cosa?
Iuto! :mc:

E' un po' in ritardo, ma spero serva da spunto di riflessione:

Premessa: credo intuitivamente (non saprei dimostrarlo) che il punto P che abbia la massima distanza da un'insieme di punti dati all'interno di una circonferenza appartenga alla stessa circonferenza

devi trovare le coordinate del punto P tali che la somma delle distanze del punto P dagli altri punti dati (A, B, C, ...) sia massima E tali che il punto P appartenga alla circonferenza data.

in pratica si tratta di sommare tutte le distanze AP, BP, CP, ecc e trovare il massimo della funzione ottenuta (derivata prima uguale a 0, derivata seconda minore di 0). Il risultato trovato (del tipo x = f(y) o y = f(x)) lo metti a sistema coll'equazione della circonferenza e trovi il risultato.

Se invece non è vero che il punto P si trova necessariamente sulla circonferenza devi imporre che il punto stia al di sopra della (o si trovi sulla) semicirconferenza inferiore e al di sotto di quella superiore (come sopra) quando fai il sistema

antcos 03-01-2013 17:07

Indipendenza lineare
 
Salve,
stavo studiando l'indipendenza lineare ma mi è sorto un dubbio:
http://www.mat.uniroma1.it/~garroni/pdf/Lezione5.pdf
per l'esempio 39 c'è la combinazione lineare
4x +y− 5z (ho cambiato le lettere per far capire meglio)
e il vettore risultante è (34
15)
ora se volessi fare il ragionamento inverso,e cioè trovare i coefficienti come posso fare?? perchè ho 3 incognite ma le equazioni sono solo 2:doh:

Dbz 09-01-2013 00:46

ENDOMORFISMI,AUTOVETTORI,AUTOVALORI
 
Anche io ho parecchi dubbi in algebra lineare:

In particolare,volevo che cos'è un autovettore,autovalore e autospazio una volta fissato un endomorfismo T: R^n--->R^n


Comunque,se ti posso essere d'aiuto...
Due vettori sono linearmente dipendenti se e solo se sono proporzionali tra loro...ovvero l'uno si può scrivere come combinazione lineare dell'altro...

robertogl 09-01-2013 17:34

Quote:

Originariamente inviato da Dbz (Messaggio 38832381)
Anche io ho parecchi dubbi in algebra lineare:

In particolare,volevo che cos'è un autovettore,autovalore e autospazio una volta fissato un endomorfismo T: R^n--->R^n


Comunque,se ti posso essere d'aiuto...
Due vettori sono linearmente dipendenti se e solo se sono proporzionali tra loro...ovvero l'uno si può scrivere come combinazione lineare dell'altro...

Gli autovalori li puoi trovare ponendo il determinante della matrice (A-x) uguale a zero. (A è la matrice di cui vuoi gli autovalori, e sottrari x agli elementi in diagonale). Poni il determinante che trovi uguale a zero, e i valori di x che soddisfano l'equazioni sono gli autovalori.
Gli autovettori li trovi facendo il ker della matrice A-x, ma con x l'autovalore relativo. (con più autovalori quindi avrai più autovettori)
L'autospazio è lo spazio generato dagli autovettori.

Ziosilvio 12-01-2013 12:49

Quote:

Originariamente inviato da robertogl (Messaggio 38836342)
Gli autovalori li puoi trovare ponendo il determinante della matrice (A-xI) uguale a zero. (A è la matrice di cui vuoi gli autovalori, e sottrari x agli elementi in diagonale). Poni il determinante che trovi uguale a zero, e i valori di x che soddisfano l'equazioni sono gli autovalori.
Gli autovettori li trovi facendo il ker della matrice A-xI, ma con x l'autovalore relativo. (con più autovalori quindi avrai più autovettori)
L'autospazio è lo spazio generato dagli autovettori.

Fixed ;) (I è la matrice identità)

robertogl 12-01-2013 12:55

Quote:

Originariamente inviato da Ziosilvio (Messaggio 38849135)
Fixed ;) (I è la matrice identità)

hai ragione, grazie della precisazione ;)

serbring 12-01-2013 13:56

ciao a tutti,

sto studiando gli estremi vincolati ed in particolar modo l'analisi di sensitività definita come variazione del massimo della funzione al variare della variabili.
Ad esempio:
Supponendo di avere questo problema, sono interessato a sapere come varia f(x,y) al variare di x e di y. In un problema non vincolato, basterebbe guardare gli elementi sulla diagonale della matrice hessiana, ma come si trasforma il tutto per un problema vincolato? Devo guardare gli elementi sulla diagonale dell'hessiano orlato?

Grazie

Dbz 20-01-2013 18:01

AUTOVETTORI E AUTOVALORI
 
Quote:

Originariamente inviato da robertogl (Messaggio 38836342)
Gli autovalori li puoi trovare ponendo il determinante della matrice (A-x) uguale a zero. (A è la matrice di cui vuoi gli autovalori, e sottrari x agli elementi in diagonale). Poni il determinante che trovi uguale a zero, e i valori di x che soddisfano l'equazioni sono gli autovalori.
Gli autovettori li trovi facendo il ker della matrice A-x, ma con x l'autovalore relativo. (con più autovalori quindi avrai più autovettori)
L'autospazio è lo spazio generato dagli autovettori.

Credo di aver capito:

Quindi dato un endomorfismo T:V-->V corpo:K

Si definiscono autovettori i vettori tali che:

T(v)=av con a appartenente a K (AUTOVALORE)
con v appartenente a V (AUTOVETTORE)

Quindi in senso euclideo stretto: sono i vettori che in seguito ad una trasformazione lineare non vengono "ruotati"
quindi :
il vettori av e v sono linearmente dipendenti

Ovviamente il vettore nullo non può essere autovettore

xxxyyy 25-01-2013 16:59

Qualcuno sa cos'e' la formula di Plemey?

Ziosilvio 25-01-2013 21:00

Quote:

Originariamente inviato da xxxyyy (Messaggio 38920758)
Qualcuno sa cos'e' la formula di Plemey?

Credo sia un teorema di analisi complessa:
http://en.wikipedia.org/wiki/Sokhotski–Plemelj_theorem

xxxyyy 26-01-2013 01:52

Quote:

Originariamente inviato da Ziosilvio (Messaggio 38921950)
Credo sia un teorema di analisi complessa:
http://en.wikipedia.org/wiki/Sokhotski–Plemelj_theorem

perfetto. Plemelj allora...
Grazie
:)

serbring 26-01-2013 10:34

Quote:

Originariamente inviato da serbring (Messaggio 38849534)
ciao a tutti,

sto studiando gli estremi vincolati ed in particolar modo l'analisi di sensitività definita come variazione del massimo della funzione al variare della variabili.
Ad esempio:
Supponendo di avere questo problema, sono interessato a sapere come varia f(x,y) al variare di x e di y. In un problema non vincolato, basterebbe guardare gli elementi sulla diagonale della matrice hessiana, ma come si trasforma il tutto per un problema vincolato? Devo guardare gli elementi sulla diagonale dell'hessiano orlato?

Grazie

nessuno sà aiutarmi?

barzi 28-01-2013 21:51

Derivate e costanti di Lipschitz
 
Ciao a tutti,

ho un problema che non riesco a risolvere.

Sia f:R^n -> R^n una funzione continuamente differenziabile. Sia g_i la sua derivata parziale rispetto a un argomento, e.g. g_i(x_1,...,x_i,...x_n)=∂/∂x_i f(x_1,...,x_i,...x_n).
Posso scrivere

||g_i(x_1,...,x_i,...,x_n)-g_i(x_1,...,y_i,...,x_n)|| ≤ L_i ||f_i(x_1,...,x_i,...,x_n)-f_i(x_1,...,y_i,...,x_n)||

dove L_i è un appropriata costante di Lipschitz? Se si, L_i è la costante di Lipschitz della funzione g_i rispetto a x_i o della funzione f rispetto a x_i? Posso prendere L_i come il massimo valore della derivata (di f oppure di g) rispetto alle variabili x_1,..., x_n?

Grazie 1000! :)

Ziosilvio 29-01-2013 08:14

Quote:

Originariamente inviato da barzi (Messaggio 38937160)
Ciao a tutti,

ho un problema che non riesco a risolvere.

Sia f:R^n -> R^n una funzione continuamente differenziabile. Sia g_i la sua derivata parziale rispetto a un argomento, e.g. g_i(x_1,...,x_i,...x_n)=∂/∂x_i f(x_1,...,x_i,...x_n).
Posso scrivere

||g_i(x_1,...,x_i,...,x_n)-g_i(x_1,...,y_i,...,x_n)|| ≤ L_i ||f_i(x_1,...,x_i,...,x_n)-f_i(x_1,...,y_i,...,x_n)||

dove L_i è un appropriata costante di Lipschitz? Se si, L_i è la costante di Lipschitz della funzione g_i rispetto a x_i o della funzione f rispetto a x_i? Posso prendere L_i come il massimo valore della derivata (di f oppure di g) rispetto alle variabili x_1,..., x_n?

Grazie 1000! :)

No, in generale non puoi.
Come controesempio veloce, prendi n=1 e g(x) = x^3: allora g'(x) = 3x^2 è continua ma non lipschitziana in R.

barzi 29-01-2013 09:25

Quote:

Originariamente inviato da Ziosilvio (Messaggio 38938146)
No, in generale non puoi.
Come controesempio veloce, prendi n=1 e g(x) = x^3: allora g'(x) = 3x^2 è continua ma non lipschitziana in R.

Hum, hai ragione. Non c'e un modo per trovare un bound di ||g_i|| che dipenda da ||f|| in genere? Oppure, in che casi si puó trovare? Thanks again! :)

barzi 29-01-2013 11:57

...altra domanda, sempre sul fatto di trovare un bound.
Se ho un eq. differenziale nella forma:

\dot x = f(x)

il seguente ragionamento é giusto?

d/dt \dot x = \partial f / \partial x \dot x = \partial f / \partial x f(x)

da cui, prendendo la norma in ambo i membri segue

||d/dt \dot x|| = ||\partial f / \partial x \dot x ||= ||\partial f / \partial x f(x)|| \le ||\partial f / \partial x f(x)|| ||f(x)|| \le L ||f(x)||

dove L é la costante di Lipschitz di f rispetto a x.
Quindi posso concludere che

||d/dt f(x)|| \le L ||f(x)|| ????

Grazie! :)

Ziosilvio 29-01-2013 12:40

Quote:

Originariamente inviato da barzi (Messaggio 38939672)
...altra domanda, sempre sul fatto di trovare un bound.
Se ho un eq. differenziale nella forma:

\dot x = f(x)

il seguente ragionamento é giusto?

d/dt \dot x = \partial f / \partial x \dot x

Direi di no, perché a primo membro hai una derivata seconda rispetto a t, e a secondo membro hai una derivata seconda in croce rispetto a t ed x.

barzi 29-01-2013 12:47

Quote:

Originariamente inviato da Ziosilvio (Messaggio 38939955)
Direi di no, perché a primo membro hai una derivata seconda rispetto a t, e a secondo membro hai una derivata seconda in croce rispetto a t ed x.

Hum... non capisco. Quello che ho scritto non deriva dalla definizione di derivata di Lie?
Ad esempio:



Non va bene?

P.s. x é funzione di t, i.e. x(t)

Ziosilvio 29-01-2013 21:42

Quote:

Originariamente inviato da barzi (Messaggio 38939672)
...altra domanda, sempre sul fatto di trovare un bound.
Se ho un eq. differenziale nella forma:

\dot x = f(x)

il seguente ragionamento é giusto?

d/dt \dot x = \partial f / \partial x \dot x

Quote:

Originariamente inviato da Ziosilvio (Messaggio 38939955)
Direi di no, perché a primo membro hai una derivata seconda rispetto a t, e a secondo membro hai una derivata seconda in croce rispetto a t ed x.

Quote:

Originariamente inviato da barzi (Messaggio 38939998)
Hum... non capisco. Quello che ho scritto non deriva dalla definizione di derivata di Lie?
Ad esempio:



Non va bene?

P.s. x é funzione di t, i.e. x(t)

Aspetta che rivedo...

Nel tuo primo post, tu scrivi:



e poi



Nel secondo, scrivi:



A me, adesso, 'ste due cose sembrano uguali... il che vuol dire che, a cose riviste, hai ragione tu.
Mi sa che mi sono impapocchiato con la notazione LaTeX...

barzi 30-01-2013 08:22

Quote:

Originariamente inviato da Ziosilvio (Messaggio 38943544)
Aspetta che rivedo...

Nel tuo primo post, tu scrivi:



e poi



Nel secondo, scrivi:



A me, adesso, 'ste due cose sembrano uguali... il che vuol dire che, a cose riviste, hai ragione tu.
Mi sa che mi sono impapocchiato con la notazione LaTeX...

Ciao, nessun problema, forse sono io che non sono stato abbastanza chiaro. Comunque grazie mille. L'importante é che ció che ho scritto sia corretto. Ora ho un'ultimissima domanda. Dal mio ragionamento sopra, posso anche dire che



dove
???

Come posso mostrarlo passo-passo?

Grazie ancora! :)

Ziosilvio 30-01-2013 10:53

Quote:

Originariamente inviato da barzi (Messaggio 38944694)
Ciao, nessun problema, forse sono io che non sono stato abbastanza chiaro. Comunque grazie mille. L'importante é che ció che ho scritto sia corretto. Ora ho un'ultimissima domanda. Dal mio ragionamento sopra, posso anche dire che



dove
???

Come posso mostrarlo passo-passo?

Grazie ancora! :)

Ehm... per quale motivo il modulo della derivata prima dovrebbe avere un massimo?, o anche solo un estremo superiore finito?
Non mi pare che tu abbia ipotesi di, che so, compattezza...

barzi 30-01-2013 11:30

Quote:

Originariamente inviato da Ziosilvio (Messaggio 38945644)
Ehm... per quale motivo il modulo della derivata prima dovrebbe avere un massimo?, o anche solo un estremo superiore finito?
Non mi pare che tu abbia ipotesi di, che so, compattezza...

Hai ragione, diciamo anche che f é C1 su un compatto D. :)

Peró non so come dimostrare il punto del post precedente :( Potresti aiutarmi? :)

psico88 03-02-2013 20:13

Ciao a tutti, vorrei una mano su un paio di esercizi che mi hanno messo in confusione...



Risposta corretta indicata: a
Allora, che d ed e siano sbagliati non c'è dubbio. Il massimo non c'è perché il valore 3 è il valore al limite e non verrà mai raggiunto, quindi b è sbagliato. Di sicuro A è limitato, quindi a è corretto. Ma scusate, perché c è sbagliato? Togliendo n = 0 che è escluso (e per cui non sarebbe neanche definita) e n = 1 per cui x = 0 (anch'esso escluso), il minimo lo ottengo subito con n = 2, e vale 3/2, che è incluso in A :confused:

robertogl 03-02-2013 20:16

edit

psico88 03-02-2013 20:22

Quote:

Originariamente inviato da robertogl (Messaggio 38970628)
Con n=1, resta 3-3=0 che però non è incluso nell'intersezione.

sì scusa volevo scrivere n = 2, ho corretto dopo. Ok n = 1 non è incluso nell'intersezione, quindi il minimo ce l'avrei subito per n = 2 a questo punto, no? Cioè, voglio dire, se la "funzione" fosse definita sui numeri reali invece degli interi allora direi che è vero, perché lo zero essendo fuori non verrà mai raggiunto, ma la funzione ci andrà "vicinissima", e sarà quindi l'estremo inferiore. Ma con i naturali, tra n = 1 e n = 2 non c'è nulla, quindi non va "vicinissima" allo zero, ma il primo valore che c'è è quello per n = 2 appunto, che mi viene da dire sia il minimo. Non so se mi sono spiegato :stordita:

robertogl 03-02-2013 20:29

Quote:

Originariamente inviato da psico88 (Messaggio 38970661)
sì scusa volevo scrivere n = 2, ho corretto dopo. Ok n = 1 non è incluso nell'intersezione, quindi il minimo ce l'avrei subito per n = 2 a questo punto, no? Cioè, voglio dire, se la "funzione" fosse definita sui numeri reali invece degli interi allora direi che è vero, perché lo zero essendo fuori non verrà mai raggiunto, ma la funzione ci andrà "vicinissima", e sarà quindi l'estremo inferiore. Ma con i naturali, tra n = 1 e n = 2 non c'è nulla, quindi non va "vicinissima" allo zero, ma il primo valore che c'è è quello per n = 2 appunto, che mi viene da dire sia il minimo. Non so se mi sono spiegato :stordita:

Si con n=2 assume minimo in 3\2, quindi io direi che anche c è vera, sarei d'accordo con te :stordita:

IngMetallo 04-02-2013 19:33

Guardando quell'insieme io direi che non è limitato.

La prima parte dell'intersezione contiene infiniti elementi partendo da 3/2 che è il minimo. L'estremo superiore dell'insieme è 3. Tutti gli infiniti elementi sono compresi tra 3/2 e 3.
Facendo l'intersezione con gli x reali che verificano : 0<x<4, l'insieme rimane comunque non limitato.

Quindi la risposta (a) direi che è sbagliata. Possibile oppure ho detto qualche castroneria ?

robertogl 04-02-2013 20:09

Quote:

Originariamente inviato da IngMetallo (Messaggio 38975937)
Guardando quell'insieme io direi che non è limitato.

La prima parte dell'intersezione contiene infiniti elementi partendo da 3/2 che è il minimo. L'estremo superiore dell'insieme è 3. Tutti gli infiniti elementi sono compresi tra 3/2 e 3.
Facendo l'intersezione con gli x reali che verificano : 0<x<4, l'insieme rimane comunque non limitato.

Quindi la risposta (a) direi che è sbagliata. Possibile oppure ho detto qualche castroneria ?

Attento, limitato non significa che contiene un numero finito di elementi. Significa che posso trovare una maggiorante e un minorante all'insieme stesso.

IngMetallo 04-02-2013 20:24

Quote:

Originariamente inviato da robertogl (Messaggio 38976125)
Attento, limitato non significa che contiene un numero finito di elementi. Significa che posso trovare una maggiorante e un minorante all'insieme stesso.

Hai ragione :doh: dovrei rifarmi un ripassino di analisi :D

xxxyyy 24-02-2013 16:57

cut

thread sbagliato

barzi 06-03-2013 21:20

Edit

dr-omega 12-03-2013 09:03

[Math] Calcolo metri lineari in funzione della circonferenza
 
Ciao a tutti, è da un paio di giorni che sto pensando a come risolvere un piccolo problema matematico, solo che al momento non ne sono ancora venuto a capo.

Il problema è il seguente: io ho una bobina di carta lunga 120Mt avente una circonferenza di 513mm la cui anima (il tubo di cartone dove è avvolta la carta) ha una circonferenza di 264mm.
Io vorrei sapere quanti metri di carta dispongo via via che essa viene utilizzata, semplicemente misurando la circonferenza ogni volta.

Quindi:

C1=264mm=0Mt
C2=513mm=120Mt

Cn=(un valore compreso tra 513 e 264)mm=xxMt

Io stavo pensando che forse dovrei considerare l'area del materiale, però non sono sicuro che sia la strada giusta:

A2=area totale del rotolo di lunghezza 120Mt
A1=area del solo tubo che corrisponde a 0Mt

A2-A1=120Mt???

Se sviluppo la formula ricavo valori non corretti...:mbe:
Un aiutino....:D

Ziosilvio 12-03-2013 09:35

Unisco al thread ufficiale delle richieste di aiuto in matematica.

E credo che dovresti considerare anche lo spessore: la carta avvolta attorno al rullo si disporrà approssimativamente come una spirale...

dr-omega 12-03-2013 12:03

Quote:

Originariamente inviato da Ziosilvio (Messaggio 39169647)
Unisco al thread ufficiale delle richieste di aiuto in matematica.

E credo che dovresti considerare anche lo spessore: la carta avvolta attorno al rullo si disporrà approssimativamente come una spirale...


Grazie mille, mi era proprio sfiggito! :doh:

Credo che lo spessore della carta lo potrei richiedere al produttore, però un conto è una pila di fogli, ed un conto appunto è un "nastro" continuo di carta che si avvolge come hai detto giustamente a spirale.

dr-omega 12-03-2013 12:08

Quote:

Originariamente inviato da Antonio23 (Messaggio 39169745)
approssimativamente, credo che dovresti considerare il numero di avvolgimenti e lo spessore del rullo, se ipotizzi che ogni avvolgimento è circolare, sai quanto è ogni avvolgimento, sai quanto è spesso... dovrebbe essere una approssimazione ragionevole, imho..

Se conosco lo spessore posso risalire al numero di avvolgimenti, però ignoro il rapporto che lega questo numero alla lunghezza, in altre parole a 60metri non sarò a metà strada tra diametro bobina da 120 meno diametro tubo, ma bensì sarò circa a 2/3.

Il trucco credo sia usare l'avvolgimento a spirale...:stordita:

error 404 15-04-2013 13:20

1 Allegato(i)
Riguardo al problema degli avvolgimenti della bobina io ho risolto così:

http://www.hwupgrade.it/forum/attach...1&d=1366027599

EDIT: ovviamente si può fare una pesante semplificazione alla sommatoria (già è inutile farla partire da 0...) usando la formula di gauss per il calcolo dei primi n numeri: dopo il + si può scrivere "Stot(Stot+1)/2" al posto della sommatoria.

dove:
Stot = spessore degli avvolgimenti (escluso lo spessore della bobina e il raggio della bobina);
r0 = Sb + rb = spessore bobina + raggio bobina;
Sc = spessore carta;
L = lunghezza.

Ho risolto il problema nel discreto perché tra un avvolgimento e l'altro lo spessore risultante è la somma dello spessore dei fogli, infatti nel punto dove c'è lo scalino questo è trascurabile. A meno che tu non sia così sfortunato da misurare lo spessore proprio in corrispondenza degli scalini! ma comunque se la carta è molta, lo scalino verrà appiattito dal peso. Alla luce di ciò non ho considerato una spirale (caso continuo, ma più ostico), ma ho considerato la bobina come circonferenze concentriche con raggio che aumenta, andando verso l'esterno, di un fattore Sc = spessore carta.

Ti consiglio di farti dare l'intervallo entro cui il produttore pensa stia effettivamente lo spessore della carta, così puoi ricavarti una stima dal basso e una dall'alto.

Se vuoi posso impegnarmi a farti una funzione per il caso continuo o un programmino in java/c++ per questo discreto :D

zanardi84 18-04-2013 09:21

"Si ritiene che in un'ora arrivino 4 telefonate ad un centralino. Supponendo che arrivino a caso secondo una poissoniana, trovare la probabilità che arrivino 2 chiamate in 2 ore."

Mi viene da pensare che le telefonate arrivino o entrambe nella prima ora (P1), o entrambe nella seconda (sempre P1), o una nella prima e una nella seconda (P2).
Posso calcolare le probabilità singolarmente applicando la formula della probabilità poissoniana.

P0 probabilità che non arrivino telefonate = e^-4
P1 cioè probabilità che arrivino 2 telefonate in un'ora = 8e^-4
P2 cioè probabilità che in un'ora arrivi una sola telefonata = 4e^-4


La soluzione indica questa formula: P = 2*e^-8*8+4e^-4*4e^-4.

Come la ottiene e perchè?

Grazie.


Tutti gli orari sono GMT +1. Ora sono le: 00:28.

Powered by vBulletin® Version 3.6.4
Copyright ©2000 - 2024, Jelsoft Enterprises Ltd.
Hardware Upgrade S.r.l.